3

In the solution manual of Boyd & Vandenberghe's Convex Optimization, we have the solution for problem 2.27. I have the following queries.

  1. Should the first sentence in the solution be "Let $H$ be the intersection of all the halfspaces that contain $C$" instead of "Let $H$ be the set of all..."?

  2. Can anybody explain more comprehensively what does the last five lines in the solution mean? It will be very helpful if a contradictory example is presented.

  3. Is it always true that if the interior of a closed set is convex then the set is also convex?

Thanks in advance.


My try to show that $C$ and $H$ have the same boundary.

Since $C$ and $H$ are both closed therefore we can write $C= int(C)\cup bd(c), H=int(H)\cup bd(H)$ (where $int$ means interior and $bd$ means boundary). Now I try to show that $bd(C)=bd(H)$ by first showing that $bd(C)\subseteq bd(H)$ and then showing that $bd(H)\subseteq bd(C)$.

  1. If $bd(C)\subseteq bd(H)$ then it means that any $x\in bd(C)$ should also be in $bd(H)$ i.e. $x\in bd(C)$ implies $x \in bd(H)$. If $x \in bd(C)$ and if it is not in $bd(H)$ then it should be either in the complement of $H$ or in the $int(H)$. The first possibility is not valid since any element of $C$ (and hence any element in $bd(C)$) must be in $H$ because $C \subseteq H$. Now the second possibility is that $x\in int(H)$. If this is true then it means that there is some $a^T$ such that $a^Tx<\sup_{y\in H}\{a^Ty\}$. This actually means that we can have a hyperplane with $a^Ty'<a^Ty$ and the halfspace associated with the $a^Ty'$ also contains $C$. But the intersection of this new halfspace (associated with $a^Ty'$) and the halfspace associated with $a^Ty$ is a set which is smaller than the halfspace associated with $a^Ty$. And therefore the intersection of all the other halfspaces that contains $C$ and this new halfspace (associated with $a^Ty'$) results in a smaller set as compared to the original intersection of all the other halfspaces and the halfspace associated with $a^Ty$. The new intersection also contains $C$. Therefore, we conclude that our original choice of $a^Ty$ can be replaced with $a^Ty'$ (while changing $y'$ until we have $y'=x$). Since the points that lie in between $y'$ and $y$ are not in the set $H$ therefore $y'=x$ is a boundary point of $H$.

  2. If $bd(H)\subseteq bd(C)$ then it means that $x\in bd(H)$ implies $x\in bd(C)$. Again we can check the two possibilities i.e. i- is it possible that $x\in bd(H)$ is in interior of $C$ and ii- is it possible that $x\in bd(H)$ and $x\in C^c$. The first possibility is not valid since it means that $C \nsubseteq H$. We check the second possibility. Using the reasoning provided in point 1 it can be shown that this case is also invalid. So the only set that $x$ can lie in is $bd(C)$.

Therefore, we have $bd(H)\subseteq bd(C)$ and $bd(C)\subseteq bd(H)$ which means $bd(C)=bd(H)$. Please let me know what is wrong with the above reasoning.

Frank Moses
  • 2,718
  • The answer to your last question is no, take $A=\overline{B(0,1)} \cup {2}$, then $A$ is closed, the interior is convex but $A$ is not convex. The answer to your first question is yes. – copper.hat May 16 '18 at 05:18
  • @copper.hat Thank you for your answer. What does $\overbar{B(0,1)}$ means? Is it a ball with radius $1$ and centered at origin or what? – Frank Moses May 16 '18 at 05:22
  • the example that you gave is not convex because we do not have supporting hyperplane at all the boundary points. If we do have the supporting hyperplanes at all the boundary points and if the set is closed then the statement "If a set is close, have supporting hyperplanes at all the bundary points and interior is convex then the set is convex". Right? – Frank Moses May 16 '18 at 05:27
  • $B(0,1)$ is a ball of radius 1 centered at the origin, the overline means closure. – copper.hat May 16 '18 at 05:34
  • I need to sleep on the above proof before I could give a comfortable answer. – copper.hat May 16 '18 at 05:35
  • Consider the set consisting of two points on the real line. It is closed, has supporting hyperplanes and the interior is empty (hence convex). But the set is not convex. – Michal Adamaszek May 16 '18 at 06:37
  • @MichalAdamaszek thank you so much for your clarifying example. So if I add the non-empty interior condition in my statement then it will be correct. Right? – Frank Moses May 16 '18 at 06:46
  • 3
    @FrankMoses It is not clear that the authors would like seeing text from their solution manual here on Math.SE. Their web page says: Instructors can obtain complete solutions to exercises by email request to us; please give us the URL of the course you are teaching. This suggests they do not wish the solution manual to be freely distributed. – Michael Grant May 16 '18 at 14:04
  • It is not immediately obvious to me that $\partial C \subset \partial H$. There is a supporting hyperplane at any element of $\partial C$, and clearly the point is in the boundary of the hyperplane, but what is not clear to me is that the point is in the boundary of $H$. This, unfortunately, is the key element of the proof above. – copper.hat May 16 '18 at 17:15
  • @michael grant i removed the image. But i think it is freely available. – Frank Moses May 16 '18 at 18:02
  • @copper.hat i think H is the intersection of all the halfspaces that contain C therefore i think that any point that is in boundary of C is also in bondary of H. – Frank Moses May 16 '18 at 18:05
  • Well, it is true, but a priori it is not clear (to me) that the boundary of point of $C$ is a boundary point of $H$. I will add a different sort of answer below. – copper.hat May 16 '18 at 18:15
  • 1
    Related to https://math.stackexchange.com/q/27285/27978. – copper.hat May 16 '18 at 18:22

1 Answers1

4

This is Robert Israel's answer https://math.stackexchange.com/a/27290/27978 marginally expanded.

It is clear that $C \subset H$.

Suppose $ p \notin C$, choose $x \in C^\circ$ and let $t = \sup \{ s \in [0,1] | x+\lambda(p-x) \in C \ \forall \lambda \in [0,s]\}$.

Since $C^c$ and $C^\circ$ are open, it is clear that $t \in (0,1)$ and $b=x+t(p-x) \in \partial C$ and, by assumption, there is a supporting hyperplane $\eta$ passing through $b$.

To finish, we need to establish that $x,p$ are strictly on opposite sides of $\eta$, from which it will follow that $p \notin H$, and hence $C=H$.

I will abuse notation slightly by using $\eta$ to denote the linear functional corresponding to the separating hyperplane. In particular, choosing the sign of $\eta$ appropriately we see that $C$ is contained in the half space $\{z | \eta(z) \le \eta(b) \}$.

Since $B(x,r) \subset C$ for some $r>0$, we have $\eta(x) < \eta(b)$. Since $p = b + {1 -t \over t} (b-x)$, we see that $\eta(p) = \eta(b) +{1 -t \over t} \eta(b-x)> \eta(b)$ from which the result follows.

Addendum: Clarifications for questions in comments.

To show that $b \in \partial C$:

Let $l(\lambda) = x+\lambda(p-x)$. Note that $l(0) = x \in C^\circ$ and $l(1) = p \in C^c$. Both $C^\circ, C^c$ are open, hence there is some $\delta>0$ such that $l(\lambda) \in C^\circ$ for $\lambda \in [0,\delta)$ and similarly $l(\lambda) \in C^c$ for $\lambda \in (1-\delta,1]$. Define $t$ as above, then it is clear that $t \in [\delta,1-\delta]$.

By definition of $t$, we have $l(s) \in C$ for all $s < t$, hence $l(t) \in C$, since $C$ is closed. By definition of $t$, $l(s) \notin C$ for any $s>t$ (otherwise the definition of $t$ would be contradicted). Hence $b=l(t) \in \partial C$.

To show that $\eta(x) < \eta(b)$:

We have that the halfspace $H= \{z | \eta(z) \le \eta(b) \}$ and $C \subset H$. In particular, $\eta(x) \le \eta(b)$. Since $\eta$ is non zero, it maps open sets into open sets. Since $B(x,r) \subset C^\circ$ for some $r>0$ we see that $\eta(B(x,r)) \subset (-\infty, \eta(b)]$ and so $\eta(x) < \eta(b)$.

copper.hat
  • 172,524
  • 1
    What do mean by $C^{\circ}$ and $C^{c} $? – Frank Moses May 16 '18 at 19:54
  • 1
    $C^\circ$ is the interior, $C^c$ is the complement. – copper.hat May 16 '18 at 20:09
  • I think the above is a simpler proof than the one you posted. I am still stuck on showing the boundary inclusion statement in that proof. – copper.hat May 16 '18 at 20:13
  • Should it not be $x+t (p-x)\in \partial C $ instead? – Frank Moses May 16 '18 at 20:16
  • Assuming that H is closed and convex then it means that we can check the followin two conditions i) can an element of boundary of H be in interior of C (no it cant be) ii) can it be in complement of C (no it can not be). – Frank Moses May 16 '18 at 20:20
  • 1
    I fixed the $s,t$ thing. I don't understand your last comment. $H$ is the intersection of the enclosing halfspaces. Since it contains $C$ is must have an interior. – copper.hat May 16 '18 at 20:21
  • I did not say that H has no interior i said the boudary element of H can not be in the interior of C. Does it make some sense now? – Frank Moses May 16 '18 at 20:25
  • 1
    Sorry, I'm not sure what point you are making? It is not immediate to me that if $x $ is a boundary point of $C$ then it is a boundary point of $H$. (The result is true, as the above asserts, but I think one would need to do something like I did above to establish the result, in which case one might as well go with the simpler proof in the first place.) – copper.hat May 16 '18 at 20:28
  • @coppper.hat I have added a reasoning in my question to show that $bd(C)=bd(H)$. Please let me know if it is wrong reasoning. I will be very thankful to you. – Frank Moses May 16 '18 at 23:56
  • was that right reasoning to show that $bd(C)=bd(H)$? Please comment. – Frank Moses May 17 '18 at 03:18
  • 1
    @FrankMoses: I got lost reading 1. above where you introduced $y,y'$. – copper.hat May 17 '18 at 05:11
  • I tried to show that $y$ cannot be an element of $H$ if $a^Ty>a^Ty'$ for some $a^T$ – Frank Moses May 17 '18 at 05:57
  • BTW I will try to make it simpler – Frank Moses May 17 '18 at 06:04
  • I don't understand where $y'$ came from. Will look again in the morning. – copper.hat May 17 '18 at 06:12
  • I tried to think an example in more dimensions but I could not figure it out. But for one dimension I think I have an example which might help in visualizing what $y$ and $y'$ are. Suppose we want the intersection of all the halfspaces (in $R$) which are greater than or equal to $4$ then we can check that the intersection of the halfspace associated with $x=5$ and halfspace associated with $x=4.5$ is actually the set defined by ...... – Frank Moses May 17 '18 at 23:30
  • ... the halfspace that is associated with $x=4.5$. Although this intersection contains the set of values $\leq 4$ but this is not the intersection of all the half spaces that contain the set of values $\leq 4$. And we can reduce it till $x=4$. So you can imagine that $y$ corresponds to $5$ and $y'$ corresponds to $4.5$ in this case. I know its too verbose but unfortunately I could not put in more comprehensively. – Frank Moses May 17 '18 at 23:30
  • My comments did not help about my reasoning related to $y$ and $y'$? – Frank Moses May 18 '18 at 03:08
  • 1
    Not really, I'm not sure what you are trying to do or where they came from. – copper.hat May 18 '18 at 03:11
  • 1
    I am a little biased towards the answer above at it shows how the closedness, presence of interior and the boundary point condition come into play. – copper.hat May 18 '18 at 03:12
  • $y $ is a point on the boundary of H and $y'$ is a point on the line joining x and y. – Frank Moses May 20 '18 at 10:08
  • I can't see why if $B(x,r)\subset C$ then $\eta(x)<\eta(b)$. I get the idea that the hyperplane would pass through $x$ and $b$ and it would be elements of $B(x,r)$ strictly separated by that hyperplane. But how can it be written down with detail? – Psaro Nov 17 '23 at 13:13
  • @Psaro $\eta $ separates $C$ at the point $b$ so all of $C$ is either $\eta(x) \le \eta(b)$ or $\ge$. Choose the sign so it is the former. Since $\eta$ is an open map, the ball about $x$ is an open interval that is $\le \eta(b)$. Hence $\eta(x) < \eta(b)$. – copper.hat Nov 17 '23 at 18:29
  • @copper.hat I have a couple of questions to your answer, which I'd be really grateful for if you could answer. I've been reading your answer for some days now, but I still do not understand some parts of it. 1) Why is $b=x+t(p-x) \in \partial C$? 2) Is $\eta(b-x)>0$? Why? – psie Jan 30 '24 at 22:45